2
$\begingroup$

Let $\mathit{H}$ be a (real or complex) Hilbert space and $U:\mathit{H}\rightarrow\mathit{H}$ be a unitary operator. What conditions can be placed on $U$ to guarantee a sequence $v_n$ such that $|v_n|=1$ and ($Uv_n$,$v_n$)$\rightarrow$0 as $n\rightarrow\infty$?

$\endgroup$

3 Answers 3

6
$\begingroup$

I'll suppose this is a complex Hilbert space and you're using the convention that the inner product is linear in the first argument and conjugate-linear in the second. The set of all possible $\langle Uv, v\rangle$ for unit vectors $v$ is the numerical range of $U$. Since unitary operators are normal, the closure of the numerical range is the convex hull of the spectrum of $U$. Thus a necessary and sufficient condition is that 0 is in the convex hull of the spectrum.

$\endgroup$
2
$\begingroup$

Well, this is probably too strong for you, but if your unitary is a bilateral shift, then you have plenty of unit vectors with equality: $\langle Uv,v\rangle = 0$. (See Sz.-Nagy-Foias: Harmonic analysis of Operators on Hilbert Space for the terminology.)

A wandering subspace is a subspace $L\subset H$ for which $UL\bot L$. See the shift in $l^2(\mathbb{Z})$.

$\endgroup$
1
$\begingroup$

$1$ is an "approximate eigenvalue" of $U$ ... so $1$ is in the spectrum of $U$. If $1$ is a genuine eigenvalue, then you have $(Uv,v)=0$ for some $v$, but if not then $1$ is an approximate eigenvalue. Then $1$ should be a limit point of the spectrum.

http://en.wikipedia.org/wiki/Spectrum_(functional_analysis)#Approximate_point_spectrum

$\endgroup$
3
  • $\begingroup$ Isn't this true for $< Uv_n,v_n> \to 1$???? $\endgroup$ Mar 28, 2011 at 5:51
  • 2
    $\begingroup$ Actually, having an eigenvector is the worst possible scenario for a unitary: if $Uv= \lambda v$, then $\langle Uv,v\rangle = \lambda$. You must have misunderstood something here. $\endgroup$ Mar 28, 2011 at 6:18
  • $\begingroup$ I don't follow you. If $1$ is a genuine eigenvalue, you just get vectors such that $(Uv,v)=1$. Don't you ? And this has nothing to do with the question. $\endgroup$ Mar 28, 2011 at 7:52

Your Answer

By clicking “Post Your Answer”, you agree to our terms of service and acknowledge you have read our privacy policy.

Not the answer you're looking for? Browse other questions tagged or ask your own question.